College Physics (7th Edition)

Published by Pearson
ISBN 10: 0-32160-183-1
ISBN 13: 978-0-32160-183-4

Chapter 27 - Quantum Physics - Learning Path Questions and Exercises - Exercises - Page 936: 24

Answer

(a) so stopping potential $V_{0}=0.64375V$ (b) cut off frequency $f_{0}=8.446455\times10^{14}Hz$

Work Step by Step

Given that work function of the material $\phi_{0}=3.5eV$ wavelength of the incident $\lambda= 300nm=300\times10^{-9}m=3\times10^{-7}m$ (a) calculation of stopping potential $E=hf=\frac{hc}{\lambda}$ putting $h=6.63\times10^{-34} J.s$, $c=3\times10^{8}m/s$,$\lambda=3\times10^{-7}m$ $E=hf=\frac{6.63\times10^{-34} J.s\times3\times10^{8}m/s}{3\times10^{-7}m}=6.63\times10^{-19} J$ $1.6\times10^{-19} J$ is equal to $1eV$ $1J$ is equal to $\frac{1}{1.6\times10^{-19}}eV$ so $6.63\times10^{-19} J$ is equal to $\frac{6.63\times10^{-19}}{1.6\times10^{-19}}eV$= $4.14375eV$ Energy of incident light $E=hf=6.63\times10^{-19} J=4.14375eV$ From equation 27.7 $hf= K_{max} + \phi_{0}$ $K_{max}=hf-\phi_{0}$ putting the values of $\phi_{0}$ and we will get $hf$ we will get $K_{max}=4.14375eV-3.5eV=0.64375eV$ from equation 27.5 $K_{max}=eV_{0}$ so stopping potential $V_{0} =\frac{K_{max}}{e}=\frac{0.64375eV}{e}=0.64375V$ (b) Calculation of cutoff frequency From Equation 27.8 cut off frequency is given by $f_{0}=\frac{\phi_{0}}{h}$ $h=6.63\times10^{-34} J.s=\frac{6.63\times10^{-34} }{1.6\times10^{-19} }eV.s=4.14375\times10^{-15}eV.s$ putting $h=4.14375\times10^{-15}eV.s$ and $\phi_{0}=3.5eV$ in above equation we will get $f_{0}=\frac{3.5eV}{4.14375\times10^{-15}eV.s}=0.8446455\times10^{15}/s=8.446455\times10^{14}Hz$
Update this answer!

You can help us out by revising, improving and updating this answer.

Update this answer

After you claim an answer you’ll have 24 hours to send in a draft. An editor will review the submission and either publish your submission or provide feedback.